• Shuffle
    Toggle On
    Toggle Off
  • Alphabetize
    Toggle On
    Toggle Off
  • Front First
    Toggle On
    Toggle Off
  • Both Sides
    Toggle On
    Toggle Off
  • Read
    Toggle On
    Toggle Off
Reading...
Front

Card Range To Study

through

image

Play button

image

Play button

image

Progress

1/113

Click to flip

Use LEFT and RIGHT arrow keys to navigate between flashcards;

Use UP and DOWN arrow keys to flip the card;

H to show hint;

A reads text to speech;

113 Cards in this Set

  • Front
  • Back
Parents bring their 6-month-old son to see you. He is symmetrically less than the fifth percentile for height, weight, and head circumference. He was born at 30 weeks’ gestation and weighed 1000 g. He was a planned pregnancy, and his mother’s prenatal course was uneventful until an automobile accident initiated the labor. He was ventilated for 3 days in the intensive care unit (ICU) but otherwise did well without ongoing problems. He was discharged at 8 weeks of life. Which of the following is the mostly likely explanation for his small size?

A. Chromosomal abnormality
B. Protein-caloriemalnutrition
C. Normal ex-premie infant growth
D. Malabsorption secondary to short gut syndrome
E. Congenital hypothyroidism
C. The expected weight versus age must be modified for a preterm infant. Similarly, growth for children with Down or Turner syndrome varies from that for other children. Thus, use of an appropriate growth curve is paramount. For the child in the question, weight gain should follow or exceed that of term infants. For this premature infant, when his parameters are plotted on a “premie growth chart,” normal growth is revealed.
A 13-month-old child is noted to be at the 25th percentile for weight, the 10th percentile for height, and less than the 5th percentile for head circumference. She was born at term. She was noted to have a small head at birth, to be developmentally delayed throughout her life, and to have required cataract surgery shortly after birth. She currently takes phenobarbital for seizures. Which of the following would most likely explain this child’s small size?

A. Congenital cytomegalovirus (CMV) infection
B. Downsyndrome
C. Glycogen storage disease type II
D. Congenital hypothyroidism
E. Craniopharyngioma
A. The developmental delay, intrauterine growth retardation (including microcephaly), cataracts, seizures, hepatosplenomegaly, prolonged neonatal jaundice, and purpura at birth are consistent with a congenital cytomegalovirus (CMV) or toxoplasmosis infec- tion. Calcified brain densities of CMV typically are found in a periventricular pattern; in toxoplasmosis, they are found scattered throughout the cortex.
A 2-year-old boy had been slightly less than the 50th percentile for weight, height, and head circumference, but in the last 6 months he has fallen to slightly less than the 25th percentile for weight. The pregnancy was normal, his development is as expected, and the family reports no psychosocial problems. The mother says that he is now a finicky eater (wants only macaroni and cheese at all meals), but she insists that he eat a variety of foods. The meals are marked by much frustration for everyone. His examination is normal. Which of the fol- lowing is the best next step in his care?

A. Sweat chloride testing
B. Examination of the eyes for retinal hemorrhages
C. Reassurance and counseling for family about childhood normal
developmental stage
D. Testing of stool for parasites
E. Magnetic resonance imaging (MRI) of the brain
C. Between 18 and 30 months of age children often become “picky eaters.” Their growth rate can slow, and the period can be distressing for families. Calm counseling of parents to provide nutrition, avoid “force-feeding,” and avoid providing snacks is usually effective. Close follow-up is required.
A 4-month-old child has poor weight gain. Her current weight is less than the 5th percentile, height about the 10th percentile, and head circumference at the 50th percentile. The planned pregnancy resulted in a normal, spontaneous, vaginal delivery; mother and child were dis- charged after a 48-hour hospitalization. Feeding is via breast and bot- tle; the quantity seems sufficient. The child has had no illness. The examination is unremarkable except for the child’s small size. Screening laboratory shows the hemoglobin and hematocrit are 11 mg/dL and 33%, respectively, with a platelet count of 198,000/mm3. Serum electrolyte levels are sodium 140, chloride 105, potassium 3.5, bicarbon- ate 17, blood urea nitrogen 15, and creatinine 0.3. Liver function tests are normal. Urinalysis reveals a pH of 8 with occasional epithelial cells but no white blood cells, bacteria, protein, ketones, or reducing substances. Which of the following is the most appropriate therapy for this child?

A. Transfusion with packed red blood cells (PRBCs)
B. Intravenous (IV) infusion of potassium chloride
C. Sweat chloride analysis
D. Growth hormone determination
E. Oral supplementation with bicarbonate
E. The patient has evidence of renal tubular acidosis (probably dis- tal tubular), a well-described cause of FTT. Upon confirmation of the findings, oral bicarbonate supplementation would be expected to correct the elevated chloride level, the low bicarbonate and potas- sium levels (although potassium supplements may be required), and poor growth.
A small-for-gestational age infant is born to a 35-year-old woman. He has low-set and malformed ears, microcephaly, rocker-bottom feet, inguinal hernias, cleft lip or palate, and micrognathia. Chromosomal analysis is likely to reveal which of the following?

A. Down syndrome (trisomy 21)
B. Edwards syndrome (trisomy18)
C. Holt-Oram syndrome
D. Patau syndrome (trisomy 13)
E. Turner syndrome
B. The child has trisomy 18. Other features include clenched hands with overlapping digits, small palpebral fissures, prominent occiput, short sternum, and cardiac defects (ventricular septal defect [VSD], atrial septal defect [ASD], patent ductus arteriosus [PDA], or coarc- tation of the aorta).
A 15-day-old infant has respiratory distress. A quick observation sug- gests she has slight cyanosis, hepatosplenomegaly, and features consis- tent with DS. The cardiac examination demonstrates a loud first heart sound, a wide and fixed split second heart sound, a low-pitched, mid- diastolic murmur at the lower left sternal border, and a harsh apical holosystolic murmur in the mitral area. An echocardiogram is likely to demonstrate which of the following?

A. Complete atrioventricular (AV) canal (endocardial cushion defect)
B. Hypoplastic left heart
C. Total anomalous venous return
D. Transposition of the great vessels
E. Tricuspid atresia
A. Although VSDs are common in DS, the most characteristic lesion is endocardial cushion defect (or atrioventricular [AV] canal defect). Slight cyanosis occurs because of the mixing of deoxy- genated with oxygenated blood. In the AV canal, a range of defects involving the atrial septum, the ventricular septum, and one or both of the AV valves can be seen. A complete AV canal includes ASDs and VSDs with a common AV valve. A partial AV canal includes defects of the atrial septum and separate mitral and tricuspid valve orifices.
A small-for-gestational age, dysmorphic newborn infant has micro- cephaly and sloping forehead, cutis aplasia (missing portion of the skin and hair) of the scalp, polydactyly, microphthalmia, and omphalocele. Which of the following is the most likely diagnosis?

A. Down syndrome (trisomy 21)
B. Edwards syndrome (trisomy18)
C. Holt-Oram syndrome
D. Patau syndrome (trisomy 13)
E. Turner syndrome
D. The appearance of cutis aplasia and polydactyly suggests trisomy 13. Other common features include holoprosencephaly (failure of growth of the forebrain), cleft lip or palate, postaxial polydactyly, flexed and overlapping fingers, coloboma, and cardiac defects (VSD, ASD, PDA, dextrocardia).
The parents of an 8-year-old DS boy arrive for his annual well-child visit. He wants to participate in sports, including the Special Olympics. Until further evaluation can be completed, which of the following sports would you suggest as being safe?

A. Diving
B. Football
C. Tennis
D. Tumbling
E. Wrestling
C. Until lateral cervical flexion–extension films confirm normal anatomy, contact sports and other activities that may result in forceful flexion of the neck should be avoided.
An institutionalized male juvenile delinquent upon close examination has severe nodulocystic acne, mild pectus excavatum, large teeth, prominent glabella, and relatively long face and fingers. His family says he has poor fine motor skills (such as penmanship), an explosive temper, and a low–normal IQ.

A. Fragile X syndrome
B. Klinefelter syndrome(XXY)
C. Turner syndrome (XO)
D. XXX syndrome
E. XYY male
E. XYY-affected males often have explosive tempers. Other findings include long and asymmetrical ears, increased length versus breadth for the hands, feet, and cranium, and mild pectus excavatum. By age 5 to 6 years, they tend to be taller than their peers and begin displaying aggressive or defiant behavior.
A tall, thin 14-year-old adolescent male has no signs of puberty. He was delayed in his speech development and always has done less well in school than his siblings. He is shy, and teachers report his activity is immature. Physical examination reveals breast development, and long limbs with a decreased upper segment–lower segment ratio. He has small testes and phallus.

A. Fragile X syndrome
B. Klinefelter syndrome(XXY)
C. Turner syndrome (XO)
D. XXX syndrome
E. XYY male
B. With Klinefelter syndrome, testosterone replacement allows for more normal adolescent male development, although azoospermia is the rule; the breast cancer incidence approaches that of women.
A 15-year-old adolescent female with primary amenorrhea is noted to be well below the fifth percentile for height. She has hypertension, a low posterior hairline, prominent and low-set ears, and excessive nuchal skin.

A. Fragile X syndrome
B. Klinefelter syndrome(XXY)
C. Turner syndrome (XO)
D. XXX syndrome
E. XYY male
C. Turner syndrome also includes widely spaced nipples and broad chest; cubitus valgus (increased carrying angle of arms); edema of the hands and feet in the newborn period; congenital heart disease (coarctation of the aorta or bicuspid aortic valve); horseshoe kidney; short fourth metacarpal and metatarsal; hypothyroidism; and decreased hearing. Mental development usually is normal.
A 7-year-old boy with MR was born at home at 26 weeks’ gestation to a 28-year-old mother who had received no prenatal care. An evaluation is likely to suggest his MR is related to which of the following?

A. Brain tumor
B. Chromosomal aberration
C. Complications of prematurity
D. Congenital infection with cytomegalovirus
E. Elevated serum lead levels
C. Prematurity, especially when earlier than 28 weeks’ gestation, is associated with complications (such as intraventricular hemorrhage) that can result in developmental delay and low IQ.
A 2-day-old infant has significant nasal and rectal bleeding. He was delivered by a midwife at home; the pregnancy was without complica- tions. His Apgar scores were 9 at 1 minute and 9 at 5 minutes. He has breast-fed well and has not required a health-care professional visit since birth. Which of the following vitamin deficiencies might explain his condition?

A. Vitamin A
B. VitaminB1
C. Vitamin C
D. Vitamin D
E. VitaminK
E. Newborn infants have a relative vitamin K deficiency, especially if they are breast-fed; most infants are given vitamin K at birth to prevent deficiency-related bleeding complications.
A 6-month-old infant has been growing poorly. His parents have changed his formula three times without success. His examination is remarkable for a pale, emaciated child with little subcutaneous fat and anterior fontanelle fullness. His laboratory test results are notable for a hemolytic anemia and prolonged bleeding times. Which of the fol- lowing is the most appropriate next step?

A. Gather urine for pH and electrolytes.
B. Measure serum factor IX levels.
C. Measure serum immunoglobulins.
D. Obtain a sweat chloride concentration.
E. Perform a hemoglobin electrophoresis.
D. The patient appears to have failure to thrive, with deficiencies of vitamin K (bleeding problems), vitamin A (fontanelle fullness), and vitamin E (hemolytic anemia). Cystic fibrosis (associated with vita- min malabsorption) would explain the condition.
An exclusively breast-fed infant with poor routine care is switched at 6 months of age to whole milk and table foods. Screening laboratories at 9 months of age demonstrate the hemoglobin and hematocrit to be 8 mg/dL and 25%, respectively, and the lead level to be less than 2 μg/dL. A follow-up complete blood count (CBC) 2 weeks later shows the hemoglobin to be at 7.8 mg/dL, the hematocrit 25%, the mean corpus- cular volume (MCV) 62%, and the platelet count to be 750,000/mm3. Which of the following would be the next step in the management of this child?

A. Order a hemoglobin electrophoresis.
B. Obtain a bone marrow aspiration.
C. Initiate iron supplementation.
D. Refer to a pediatric hematologist.
E. Initiate soybean-based formula.
C. The child in the question likely did not get iron (or vitamin D) supplementation in the first 6 months of life while exclusively breast- feeding and was switched to whole milk (low in iron) and to table foods (not supplemented with iron as are baby foods) at too young an age. All of the laboratory data are consistent with iron deficiency ane- mia; iron supplementation in this child with a resultant brisk erythro- cyte response is both diagnostic and therapeutic. Failure of the child to respond to the iron therapy would require further evaluation.
A 3-week-old is admitted for failure to thrive, diarrhea, and a sepsis- like picture. He does well on intravenous fluids; when begun on rou- tine infant formula with iron, his symptoms return. It is Saturday and the state health department laboratory is closed. You should begin feeds with which of the following?

A. Amino acid–based formula (Nutramigen or Pregestimil)
B. Low-phenylalanine formula (Lofenalacor Phenex-1)
C. Low-iron,routine infant formula (Similac with low iron or Enfamil with low iron)
D. Low-isoleucine,low-leucine,low-valine infant formula (Ketonex-1 or MSUD 1)
E. Soy-based formula (ProSobee or Isomil)
E. This patient appears to have galactosemia; uridyl transferase defi- ciency is the cause and the condition results in features of jaundice, hepatosplenomegaly, vomiting, hypoglycemia, seizures, lethargy, irri- tability, poor feeding and failure to thrive, aminoaciduria, liver failure, mental retardation, and an increased risk of E coli sepsis. Children with galactosemia are managed with a lactose-free formula. The low- phenylalanine formulas are for infants with phenylketonuria; low-iron formulas serve no purpose other than causing iron deficiency anemia; the low-isoleucine, low-leucine, low-valine infant formulas are useful for patients with maple syrup urine disease (MSUD); and the amino acid–based formulas are excellent for children with malabsorption syndromes.
A 14-month-old child has lower-extremity bowing, a waddling gait, genu varum, and is at the fifth percentile for height. Laboratory data include low–normal serum calcium, moderately low serum phosphate, and elevated serum alkaline phosphatase levels, hyperphosphaturia, and normal parathyroid levels. Which of the following is the most likely diagnosis?

A. Fanconi syndrome
B. Genetic primary hypophosphatemia
C. Malabsorption of vitamin D
D. Phosphate malabsorption
E. Renal osteodystrophy
B. Lower-extremity bowing, low–normal calcium and phosphate lev- els, and normal parathyroid hormone levels suggest familial primary hypophosphatemia.
An 8-month-old African-American baby arrives to the emergency department with his mother with the complaint of decreased left arm movement. He is the product of a normal term pregnancy, has had no medical problems, and was in good health when his mother dropped him off at the day care center. Upper arm radiographs show a left humerus spiral fracture. Which of the following is the most appropri- ate next step in management?

A. Admit the child and call child protective services.
B. Obtain serum 1,25(OH)2D levels.
C. Order serum alkaline phosphatase levels.
D. Obtain stool for analysis for fat-soluble vitamins.
E. Send chromosome sample for osteogenesis imperfecta analysis.
A. A spiral fracture of the humerus is suspicious but not diagnostic for child abuse. While further laboratory testing is appropriate, the next step in the management of this child is to provide a safe environment until more data are available.
The diet of a 3-year-old child with cystic fibrosis should be supple- mented with which of the following?

A. Folate
B. Sodium
C. Vitamin C
D. Vitamin B12
E. VitaminD
E. In addition to pancreatic enzyme replacement therapy, supple- mentation with fat-soluble vitamins (A, D, E, and K), often iron, and sometimes zinc is recommended.
A 5-year-old girl is somewhat short and has mild leg bowing. Her med- ical history is significant only for well-controlled seizure disorder. Serum calcium, phosphorus, and alkaline phosphatase levels and uri- nary amino acid concentration are normal. A bone age is notable for abnormal distal radius and ulna mineralization. Which of the following is the most likely diagnosis?

A. Cystic fibrosis
B. Fanconi
syndrome
C. Genetic primary hypophosphatemia
D. Rickets associated with anticonvulsive drug use
E. Schmid metaphyseal dysplasia
E. All of the rickets syndromes present with elevated alkaline phos- phatase levels. Schmid metaphyseal dysplasia, an autosomal domi- nant condition, presents in a similar way with short stature, leg bowing, and waddling gait. Radiographs show irregular long bone mineralization. Biochemically, Schmid-type metaphyseal dysostosis presents with normal serum calcium, phosphorus, and alkaline phosphatase activity and normal urinary amino acid levels.
A healthy, well-developed 6-year-old child arrives as a new patient to your clinic. His immunization card reveals that he received an immu- nization at birth and some when he was 2 months old, but none since. Which of the following statements about him is correct?

A. He should receive the live oral poliovirus vaccine rather than the inactivated (injectable) poliovirus vaccine (IPV).
B. The pertussis vaccine is contraindicated at his age and is replaced with the tetanus-diphtheria vaccine.
C. He is too old to receive the H influenzae type b vaccine.
D. His vaccinations at birth and 2 months are repeated because too
much time has elapsed for them to be effective.
E. He is too young for the varicella vaccine.
C. The Hib vaccine generally is not recommended for children 5 years of age or older.
Appropriate advice for a mother of a 2-week-old child here for a “well- child” visit includes which of the following?

A. By age 1 month the child should be sleeping through the night.
B. Children should be able to roll over by age 2 months and to sit by age 4 months.
C. Half-strength fruit juices can be initiated at age 2 months.
D. Potty training should begin at age 1 year.
E. Sleep in the supine position is recommended.
E. Juices (undiluted) are avoided until approximately 6 months of age (in a cup and not in the bottle). At 1 month of age an infant should be able to fix and follow, but not be expected to sleep through the night by age 2 to 3 months. Realistic targets for development include rolling over at 4 months and sitting by 6 months. Potty training starts when the child shows interest, usually no earlier than age 2 years. Parents are told to place healthy children on their backs (or side) for sleep to reduce the incidence of sudden infant death syndrome.
During a “well-child” visit, the parents of a healthy 5-month-old offer a great amount of information. Which of the following bits of information is of most concern?

A. A diet that includes baby cereal, five different baby vegetables,
and one baby fruit
B. Consuming 32 oz of infant formula per day
C. Intermittent tugging on the ears
D. Limited eye contact with parents
E. Rolling from front to back but not back to front
D. Children fix and follow on the human face from birth. A 5-month- old child who does not engage in eye contact is abnormal.
Which of the following statements about “routine” screening tests is accurate?

A. All children undergo tuberculosis skin testing at age 12 months.
B. Lead testing is obtained on all 12 and 14 month old infants.
C. Pelvic examinations are part of the examination of a sexually
active adolescent.
D. Screening hematocrit levels are obtained on all infants at age
2 months.
E. Universal cholesterol screening begins at age 11months.
C. Tuberculosis and lead testing are performed on at-risk children. Pelvic examinations are performed when girls become sexually active or by age 18 to 21 years. Screening hematocrits are done at age 9 to 12 months, and cholesterol tests are done for children with familial risk factors.
A girl is born via stat cesarean section to a 34-year-old mother whose pregnancy was complicated by hypertension and abnormal fetal heart monitoring. At delivery she is covered in thick, green meconium and is limp, apneic, and bradycardic. Which of the following is the best first step in her resuscitation?

A. Administer IV bicarbonate.
B. AdministerIVnaloxone.
C. Initiate bag-and-mask ventilation.
D. Initiate chest compressions immediately.
E. Intubate with an endotracheal tube and suction meconium from
the trachea.
E. An attempt is made to remove the meconium from the oropharynx and the airway prior to initiation of respirations. Ideally, the obstetri- cian will begin suctioning the meconium upon delivery of the head, and the pediatrician will further remove meconium with an aspirator or through endotracheal intubation with suction. Ventilation is initi- ated after meconium is removed. The goal is to remove airway meconium and to prevent its aspiration into the small airways where ventilation-perfusion mismatch may occur with deleterious effects.
A term male is delivered vaginally to a 22-year-old mother. Immediately after birth he is noted to have a scaphoid abdomen, cyanosis, and respi- ratory distress. Heart sounds are heard on the right side of the chest, and the breath sounds seem to be diminished on the left side. Which of the following is the most appropriate next step in his resuscitation?

A. Administer IV bicarbonate.
B. AdministerIVnaloxone.
C. Initiate bag-and-mask intubation.
D. Initiate chest compressions immediately.
E. Intubate with an endotracheal tube.
E. The case describes diaphragmatic hernia. As a result of herniated bowel contents into the chest, these children often have pulmonary hypoplasia. Bag-and-mask ventilation will cause accumulation of bowel gas (which is located in the chest) and further respiratory compromise. Therefore, endotracheal intubation is the best course of action.
A 37-week-gestation boy is born after an uncomplicated pregnancy to a 33-year-old mother. At birth he was lethargic and had a slow HR. Oxygen was administered via bag and mask, and he was intubated; his HR remained at 40 bpm. Which of the following is the most appropri- ate next step?

A. Administer IV bicarbonate.
B. AdministerIVatropine.
C. Administer IV epinephrine.
D. Administer IV calcium chloride.
E. Begin chest compressions.
E. If the HR is still less than 60 bpm despite PPV with 100% oxygen, then chest compressions are given for 30 seconds. If the HR is still less than 60 bpm, then drug therapy (usually epinephrine) is indicated.
A term infant is born vaginally after an uncomplicated pregnancy. She appears normal but has respiratory distress when she stops crying. When crying she is pink; when not she makes vigorous respiratory efforts but becomes dusky. Which of the following is the likely explanation for her symptoms?

A. Choanal atresia
B. Diaphragmatichernia
C. Meconium aspiration
D. Neonatal narcosis
E. Pneumothorax
A. Infants are obligate nose breathers. When crying they can breathe through their mouth, but they must have a patent nose when quiet. Choanal atresia is identified by passing a feeding tube through each nostril or by identification of mist on cold metal held under the infant’s nose. Should choanal atresia be diagnosed, endotracheal intubation bypasses the airway obstruction until surgical repair can be completed.
A term infant requires resuscitation after a spontaneous vaginal deliv- ery. The Apgar scores at 1, 5, and 10 minutes were 2, 7, and 9, respec- tively. The mother’s medical records show that she received routine prenatal care with normal prenatal ultrasonogram, triple screen, and glucose tolerance tests. The nurse tells you that the father seemed very agitated and mentioned “suing the obstetrician if the baby does not turn out normal.” Your examination of the baby reveals no abnormalities. In counseling the family, which of the following is most appropriate?

A. Tell them that your examination findings indicate that everything is fine.
B. Tell them that the low Apgar scores at 1 and 5 minutes indicate that the baby suffered perinatal asphyxia.
C. Tell them that because the pregnancy was uncomplicated, any neurologic deficit that the baby may develop likely can be attributed to events occurring at delivery.
D. Tell them that your examination findings are reassuring, and that you will perform a careful developmental assessment at every well- child visit.
E. Avoid speaking to the parents until you have had a chance to speak with the obstetrician and to see the cord blood gas results.
D. The Apgar score at 1 minute reflects the neonatal environment immediately prior to birth; the 5-minute score correlates the infant’s response to resuscitation. The Apgar scores are not an accurate reflection of morbidity. An examination is a better indicator of the child’s outcome, but CP cannot be ruled out on the basis of a normal neonatal physical examination. A discussion of the events of delivery is best left to the obstetrician; the majority of difficult deliveries are the result of a previously unidentified antenatal insult. However, avoidance of the parents will likely only further their anxiety and may impede your efforts to provide care for the child.
A 4-year-old child with CP comes to your clinic for the first time for a routine visit. He walks with the help of leg braces and a walker, and his speech is slurred and limited to short phrases. He has never been hospitalized and he does not have swallowing problems. He began walking at age 2.5 years, and he is unable to take off his clothes and use the toilet without help. On examination you find that the boy has only minimally increased tone in the upper extremities but good fine motor coordination; he has significantly increased tone and deep tendon reflexes in the lower extremities. How would you categorize this child’s CP?

A. Mild, diplegic
B. Mild,hemiplegic
C. Moderate, diplegic
D. Moderate, quadriplegic
E. Severe,diplegic
C. In diplegia all four extremities are affected, with greater impairment of the lower extremities. As most children walk by age 14 months, this child’s motor quotient is 14 months/30 months = 0.47, which classifies him as moderately impaired.
An infant girl is born via spontaneous vaginal delivery at 28-week ges- tation because of an incompetent cervix. Which of the following fea- tures of her clinical course in the neonatal intensive care unit (ICU) is most likely to correlate with her clinical outcome 5 years from now?

A. Administration of surfactant
B. Apneaofprematurity
C. Grade IV intraventricular hemorrhage
D. Retinopathy of prematurity stage 1 on initial ophthalmologic
examination
E. Umbilical artery catheterization
C. Intraventricular hemorrhage is a complication in preterm infants. It is associated with seizures, hydrocephalus, and periventricular leukomalacia. A grade IV bleed involves the brain parenchyma, putting this child at higher risk for neurodevelopmental handicap.
The parents of a 2-year-old girl, recent immigrants from Guatemala, bring their child to you for the first time. The child was born at term after an uncomplicated pregnancy and delivery, and her neonatal course was uneventful. She sat without support at 6 months of age, pulled to a stand at 10 months, and walked at 14 months. She has a 10-word vocabulary, is able to drink from a cup, and feeds herself with a spoon. A previous child in the family died at age 5 years from “heart trouble.” On physical examination, you note lower extremity contractures, hand stiffness, somewhat coarse facial features, and hepatosplenomegaly. The child’s growth is within normal limits, and her examination is other- wise normal. Which of the following is the most appropriate next step to diagnose this child’s condition?

A. Abdominal computerized tomography (CT)
B. Brain magnetic resonance imaging (MRI)
C. Chromosomal analysis
D. Tests for a storage disorder
E. Thyroid function studies
D. The enlarged liver and spleen, the coarse facies, and the history of death in a previous child from “heart trouble” point to a storage disorder. Her joint contractures and hand stiffness may be explained by an abnormal metabolism rather than a CNS deficit as in CP.
A term infant delivered vaginally develops vomiting and abdominal distention at age 10 hours. No stool passage has been noted. An abdominal radiograph shows distended bowel loops and a “bubbly” pattern in a portion of intestine; the colon is narrow. Which of the fol- lowing should you tell the parents?

A. You would like to consult a pediatric surgeon because you suspect that their child has Hirschsprung disease.
B. The child most likely has necrotizing enterocolitis, a condition more commonly seen in premature infants. Therefore you question the child’s supposed gestational age.
C. You are concerned about the possibility of meconium ileus and would like to obtain some family history.
D. You believe that the child simply is constipated and would like to change to a soy-based formula to see whether the baby tolerates this better.
E. The child’s symptoms and radiograph findings most likely are normal.
C. Meconium ileus, inspissated meconium obstructing the distal ileum, is thought to be caused by deficiency of proteolytic enzymes. Obstruction begins in utero, resulting in underdevelopment of distal lumina. It almost always is associated with CF. Intestinal atresia and Hirschsprung disease (congenital aganglionic megacolon) cause sim- ilar clinical pictures, but the radiographic findings for this child are most consistent with meconium ileus. Necrotizing enterocolitis also causes emesis and abdominal distension but occurs primarily in extremely low-birth-weight infants (ie, <1000 g); the colon would be expected to be of normal size. Constipation is not consistent with this baby’s clinical picture or the described radiographic picture.
Appropriate clinical management of the patient in Question 16.1 includes which of the following?

A. Change from enteral to intravenous feeds; obtain genetics consultation for the next morning.
B. Change from enteral to intravenous feeds,obtain a blood culture, and initiate antibiotics.
C. Change from enteral to intravenous feeds and obtain a STAT
pediatric surgery consultation.
D. Change from cow’s milk to soy-based infant formula and continue to observe the infant.
E. Do not change your current management.
C. Meconium ileus is a surgical emergency, as volvulus and perfora- tion peritonitis are not uncommon complications.
A 10-year-old boy has a history of recurrent sinusitis and multiple episodes of pneumonia. A sweat electrolyte test result is within the normal range. Your differential diagnosis now includes atopy, primary ciliary dyskinesia, and which of the following?

A. Tuberculosis
B. Chronic granulomatous disease
C. Coccidioidomycosis
D. Cystic fibrosis
E. Severe combined immunodeficiency
D. CF cannot be ruled out (false-negative test results); the sweat chloride test should be repeated and/or other CF diagnostic modali- ties considered. Bronchiectasis and chronic sinusitis are characteris- tic of ciliary dyskinesia syndromes. If associated with visceral situs inversus, the diagnosis of Kartagener disease is given. Sinusitis is not a common complaint among patients with tuberculosis, chronic granulomatous disease, or coccidioidomycosis. In addition to severe pulmonary infections, chronic diarrhea and wasting dominate the clinical picture in children with severe combined immunodeficiency.
A 2-month-old infant arrives with bulging of his anterior fontanelle. He is fussy, refuses to nurse or to take a bottle, and has vomited once en route to the hospital. He has had no fever. Computerized tomo- graphic (CT) scan of the head is negative for trauma or tumor. In addi- tion to meningitis, your differential diagnosis should include which of the following?

A. Colic
B. Intussusception
C. Sinusitis
D. Pneumonia
E. Vitamin A excess
E. Malabsorption of fats and protein are major causes of morbidity for patients with CF. In addition to carefully planned diets, most children with CF take supplements of the fat-soluble vitamins D, E, K, and A. Disorders of vitamin A metabolism can result in pseudo- tumor cerebri (increased intracranial pressure), which cause headache, vomiting, and neurologic abnormalities. Thus, a bulging fontanelle may be the presenting sign of CF in an infant.
A 35-week-gestation infant is delivered via cesarean section because of macrosomia and fetal distress. The mother has class D pregestational diabetes (insulin-dependent, with vascular disease); her hemoglobin A1C is 20% (normal 8%). This infant is at risk for birth asphyxia, car- diac septal hypertrophy, polycythemia, and which of the following?

A. Congenital dislocated hip
B. Dacryostenosis
C. Hyaline membrane disease
D. Hyperglycemia
E. Pneumothorax
C. Infants born to mothers with poorly controlled diabetes are at risk for respiratory distress syndrome (surfactant deficiency) at later ges- tational ages than seen in infants born to mothers who do not have diabetes.
A term infant weighing 4530 g is born without complication to a mother with class A pregestational diabetes (non-insulin requiring). His initial glucose level is 30 mg/dL, but the level after he consumes 30 cc of infant formula is 50 mg/dL, and another level obtained 30 minutes later is 55 mg/dL. His physical examination is unremarkable except for his large size. Approximately 48 hours later he appears mildly jaundiced. Vital signs are stable, and he is eating well. Which of the following serum laboratory tests are most likely to help you evaluate this infant’s jaundice?

A. Total protein, serum albumin, and liver transaminases
B. Total and direct bilirubin, liver transaminases, and a hepatitis panel
C. Total bilirubin and a hematocrit
D. Total bilirubin and a complete blood count E. Total and direct bilirubin and a complete blood count with differential and platelets
C. This baby most likely has hyperbilirubinemia secondary to liver immaturity, possibly complicated by polycythemia. He should have a high level of unconjugated bilirubin and, in the absence of intra- hepatic disease, a normal conjugated (or direct) portion. While choices D and E include the correct answer, additional tests are unnecessary for this otherwise healthy-appearing infant who contin- ues to feed well. Therefore, C is the best answer to the question.
A premature infant of a class B pregestational (insulin-requiring, but without vascular disease) diabetic mother is delivered via cesarean section due to fetal distress. The mother’s axillary temperature at delivery is 98.6°F (37°C). The child has poor color and tone, no spon- taneous cry, minimal respiratory effort, and a weak pulse of 80 bpm. After endotracheal intubation, the color and tone improve a bit, but she still has perioral cyanosis and her heart rate is 90 bpm. Which of the following is the most likely cause of her persistent distress?

A. Hypocalcemia
B. Hypoglycemia C. Impaired cardiac function
D. Renal failure
E. Sepsis
C. Infants born to mothers with poorly controlled gestational diabetes are at risk for congenital heart anomalies, cardiomyopathy, septal hypertrophy, and subaortic stenosis. This child’s symptoms and the maternal diabetes history indicate a risk for cardiac problems. Sepsis can cause similar symptoms, but no risk factors for infectious disease are noted. This child is at risk for hypoglycemia, but hypoglycemia alone would less likely explain all of his symptoms.
A term infant born to a mother with class C pregestational diabetes (insulin-dependent, but without vascular disease) requires endotra- cheal intubation at delivery for poor respiratory effort, tone, and color. His initial serum glucose level is 10 mg/dL, and the level stabilizes over 36 hours with intravenous administration of glucose. On the third day of life, his physical examination is remarkable for macrosomia and a new abdominal mass. Which of the following is the most likely cause of the abdominal mass?

A. Hydronephrosis
B. Infarction of the spleen
C. Intraintestinal air
D. Liver engorgement
E. Volvulus
A. Renal vein thrombosis can present as an abdominal mass (hydronephrosis), as a complication of polycythemia in IDMs. Such infants may have gross hematuria, but microscopic hematuria is more common. Hypertension is uncommon following an acute thrombosis but may occur as a late complication. The affected kidney may recover normal function or it may atrophy. Bilateral thrombosis can lead to chronic renal failure. If this child has delayed passage of meconium, small left colon syndrome (also seen in IDM) should be considered as a cause of the abdominal mass.
Which of the following decreases the risk of neurologic damage in a jaundiced newborn?

A. Acidosis
B. Displacement of bilirubin from binding sites by drugs such as sulfisoxazole
C. Hypoalbuminemia
D. Sepsis
E. Maternal ingestion of phenobarbital during pregnancy
E. Administration of phenobarbital induces glucuronyl transferase, thus reducing neonatal jaundice. Sepsis and acidosis increases the risk of neurologic damage by increasing the blood-brain barrier’s per- meability to bilirubin. Hypoalbuminemia reduces the infant’s ability to transport unconjugated bilirubin to the liver, and similarly drugs that displace bilirubin from albumin elevate free levels of unconju- gated bilirubin in the serum.
You are about to return a telephone call to the mother of an 8-day-old infant who continues to have jaundice which was first noted on the second day of life; you are about to report to her that his latest total and direct bilirubin levels are 12.5 and 0.9 mg/dL, respectively. You look over your chart and see that he and his mother have O type blood, the direct and indirect Coombs test is negative, his reticulocyte count is 15%, and a smear of his blood reveals no abnormal cell shapes. He is bottle-feeding well, produces normal stools and urine, and has gained weight well. Which of the following diagnoses remains in your differential diagnosis?

A. Gilbert syndrome.
B. Disseminated intravascular coagulation (DIC)
C. Spherocytosis
D. Polycythemia
E. An undiagnosed blood group isoimmunization
A. Gilbert syndrome would present with a negative Coombs test, a normal (or low) hemoglobin, a normal (or slightly elevated) reticulocyte count, and prolonged hyperbilirubinemia. Red cell morphology would be abnormal in DIC and spherocytosis, polycythemia would present with an elevated hemoglobin level (that listed above is normal for a newborn), and blood group isoimmunization would present with a positive Coombs test.
The hyperbilirubinemia associated with Crigler-Najjar syndrome type I is caused by which of the following?

A. Increased production of bilirubin B. Impaired conjugation of bilirubin
C. Deficient hepatic uptake of bilirubin
D. Severe deficiency of uridine diphosphate glucuronosyltransferase
E. Glucose-6-phosphate dehydrogenase deficiency
D. Although all infants are relatively deficient in uridine diphos- phate glucuronosyltransferase, those with Crigler-Najjar syndrome type I have a severe deficiency, causing high bilirubin levels and encephalopathy. Treatment is phototherapy. Encephalopathy is rare with Crigler-Najjar syndrome type II, in which bilirubin levels rarely exceed 20 mg/dL.
A 30-hour-old full-term infant has face and chest jaundice. He is breast- feeding well and has an otherwise normal examination. His bilirubin level is 15.5 mg/dL. Which of the following is the most appropriate course of action?

A. Recommend cessation of breast-feeding for 48 hours and supple- ment with formula.
B. Start phototherapy.
C. Wait 6 hours and retest the serum bilirubin level.
D. Start an exchange transfusion.
E. No action is needed.
B. Although the etiology of the hyperbilirubinemia must be investi- gated, phototherapy should be started.
A newborn infant was born at home. At 2 days of life he is has puffy, tense eyelids, red conjunctivae, a copious purulent ocular discharge, and chemosis. Which of the following is the most likely diagnosis?

A. Chemical conjunctivitis
B. Chlamydial conjunctivitis
C. Dacryocystitis
D. Gonococcal ophthalmia
E. Pneumococcal ophthalmia
D. The time of symptom onset in a neonate with conjunctivitis can be helpful. Chemical conjunctivitis that is self-limited and presents within 6 to 12 hours of birth is the result of ocular silver nitrate or erythromycin prophylaxis irritation. Gonococcal conjunctivitis usu- ally occurs within 2 to 5 days of birth and is the most serious of the bacterial infections; prompt and aggressive topical treatment and sys- temic antibiotics can prevent serious complications such as corneal ulceration, perforation, and resulting blindness. Parents are treated for gonococcal disease to prevent a child’s reinfection. Chlamydial conjunctivitis often presents 5 to 14 days after birth and usually is treated with systemic erythromycin (in part to reduce the infant’s risk of chlamydial pneumonia at 2-3 months of age). The risks of oral erythromycin treatment must be weighed against the increased risk of hypertrophic pyloric stenosis, a condition associated with oral erythromycin use in children. Both parents of a child with chlamydial conjunctivitis also are treated.
A term 3500-g female delivered by cesarean section develops a respi- ratory rate of 70 breaths/min and expiratory grunting at 1 hour of life. She has good tone, good color, and a strong suck. Which of the fol- lowing is the most likely diagnosis?

A. Intubation and suctioning below the vocal cords
B. Administration of surfactant
C. Initiation of antibiotic therapy
D. Swallow study and upper GI series
E. Observation for a period of several hours
E. Transient tachypnea of the newborn is a respiratory condition resulting from incomplete evacuation of fetal lung fluid in full-term infants. It occurs more commonly with cesarean deliveries and usually disappears within 24 to 48 hours of life. Often no treatment is indi- cated unless the infant requires low amounts of supplemental oxygen. Antibiotics would be indicated for a child for whom pneumonia would be suspected; these children usually do not have a vigorous suck as outlined in the question. Intubation and suctioning below the vocal folds hints at meconium aspiration; the intubation would appropriately be accomplished in the delivery room and not hours later. The barium swallow and upper GI series might be helpful to identify a tracheoesophageal fistula. Exogenous surfactant is used for premature infants for whom surfactant deficiency is suspected.
A term infant is born to a 23-year-old known HIV-positive mother. The mother has been followed closely during the pregnancy, and she has been taking antiretroviral medications for the weeks prior to the delivery. Routine management of the healthy infant should include which of the following?

A. Administration of intravenous immunoglobulin to the baby to decrease the risk of perinatal HIV infection
B. Admission to the neonatal intensivecareunit for close cardiovascular monitoring
C. Beginning a course of zidovudine for the infant
D. Chest radiographs to evaluate for congenital Pneumocystis carinii
E. HIV enzyme-linked immunosorbentassay (ELISA) on the infant to determine if congenital infection has occurred
C. HIV transmission from mother to infant has decreased by more than 50% in the past 15 years, probably as a result of perinatal anti- retroviral administration to the mother and a zidovudine course to the exposed infant. Transplacentally transmitted maternal HIV anti- bodies will result in a positive neonatal ELISA; it is not a useful test for determining newborn infection. Intravenous immunoglobulin has not been shown to have a role in decreasing perinatal transmission. Healthy infants born to HIV-infected mothers do not require special monitoring or routine radiographs.
A 2150-g infant is delivered at 34-week gestation. The mother had prenatal care in Mexico and says she had no problems. Her highest temperature during labor was 100.8°F (38.2°C). The amniotic fluid had a brown-stained appearance. At birth the infant had a diffuse ery- thematous pustular rash, pallor, poor feeding, tachypnea, and cyanosis. His CBC indicates marked monocytosis. He dies at 4 hours of age, soon after initiation of antibiotics. He most likely had which of the following?

A. Congenital syphilis
B. Congenital varicella
C. Disseminated herpes
D. GBS disease
E. Listeriosis
E. Listeria is a gram-positive rod isolated from soil, streams, sewage, certain foods, silage, dust, and slaughterhouses. The food borne transmission of disease is related to soft-ripened cheese, whole and 2% milk, undercooked chicken and hot dogs, raw vegetables, and shellfish. The newborn infant acquires the organism transplacentally or by aspiration or ingestion at delivery. The mortality rate of early- onset disease is approximately 30%.
Which of the following infants most warrants home cardiorespiratory monitoring?

A. A healthy 3-month-old infant, born at term, whose weight is at the fifth percentile
B. A healthy infant, born at 29-week gestation, whose weight is at the 50th percentile
C. A 5-month old infant with a history of recurrent bouts of wheezing
D. A premature infant with recurrent apnea and bradycardia
E. A healthy term infant whose older sibling died of SIDS
D. Home cardiorespiratory monitoring has not been shown to decrease the incidence of SIDS. Monitoring is recommended for symptomatic premature infants (ie, those with apnea and bradycar- dia), but can safely be discontinued by 43 weeks post-gestational age in most cases. Monitoring may also be warranted for children with certain underlying chronic conditions, such as those with chronic lung disease. It is not recommended for the infants in choices A, B, or C. The occurrence of a genetic susceptibility to SIDS within a family is thought to be exceedingly rare.
A pregnant woman comes to you for a prenatal visit. As her family pediatrician, your advice to her should include which of the following statements about reducing the risk of SIDS?

A. Reduce the infant’s exposure to tobacco smoke, and always place the baby in the supine position when she sleeps.
B. Always keep the baby in the prone position, even while awake.
C. Administer supplemental infant vitamins.
D. Attempt to make breast milk the infant’s primary source of nutrition.
E. Protect the infant from people who are ill.
A. Although your advice to this woman might also include choices C, D, and E, these measures have not been shown to reduce the infant’s risk of SIDS.
Which of the following statements about SIDS is accurate?

A. Most victims are girls.
B. The incidence is lowest among African Americans and Native Americans.
C. Home monitoring reduces the risk.
D. Most cases are attributable to a metabolic defect.
E. It is the most common cause of death between the ages of 7 and
365 days.
E. SIDS is the most common cause of death of infants between 1 week and 1 year of age, and it more commonly affects boys and Native- American and African-American children.
The investigation of an unexpected infant death includes a history, a postmortem examination, and which of the following?

A. DNA studies
B. An arterial blood gas measurement
C. A venous blood gas measurement
D. A death scene investigation
E. Stoolstudies
D. A death scene investigation is crucial to rule out trauma, both intentional and accidental.
A 2-month-old girl with Down syndrome is noted to have a systolic and a diastolic heart murmur, and the second heart sound is split. The liver edge is palpable 4 cm below the right costal margin. Her mother reports that lately the baby has been sweaty and sometimes blue around the mouth when she nurses, and she seems to be eating less than pre- viously. Her EKG shows a superiorly oriented QRS frontal plane axis with counterclockwise depolarization pattern and right ventricular hypertrophy. Which of the following is the most likely diagnosis?

A. Atrial septal defect
B. Atrioventricular canal defect
C. Patent ductus arteriosus
D. Patent foramen ovale
E. Ventricular septal defect
B. Atrioventricular canal defect is common among children with Down syndrome. This infant’s symptoms and clinical findings are most consistent with this diagnosis. While a simple VSD is common in patients with Down syndrome, the multitude of heart murmurs and ECG findings make this answer less likely.
A 29-week-old, 1000-g boy is admitted to the neonatal intensive care unit, where he receives routine care. He does well until day 5 of life, when he develops an increased respiratory rate, mild subcostal retrac- tions, and a widened pulse pressure, but no cyanosis or increased oxygen requirement. A continuous murmur is heard along the left sternal bor- der. Chest radiography shows pulmonary vascular congestion. Which of the following medications may best relieve his symptoms?

A. Albuterol
B. Racemic epinephrine C. Indomethacin
D. Digoxin
E. Furosemide
C. A noncyanotic heart lesion is suspected in this child who has a new heart murmur without a corresponding increase in oxygen require- ments. The murmur, not heard at birth, becomes evident after the pul- monary vascular resistance falls. His age, history, and physical findings are consistent with a patent ductus arteriosus (PDA). Indomethacin or surgical closure is used to treat this condition.
A 12-month-old boy with a stable but moderate-size ventricular sep- tal defect presents to the pediatric dentist for cleaning and manage- ment of his multiple caries. Prior to the procedure, he should receive which of the following?

A. Acetaminophen
B. Amoxicillin
C. Digoxin
D. Ditropan
E. None of the above
E. The guidelines for the use of prophylactic antibiotics are updated frequently by the American Heart Association. Among those cur- rently recommended to receive antibiotic prophylactic treatment are patients for whom any heart infection would result in the highest inci- dence of adverse outcome: previous history of endocarditis, prosthetic valve or material for repair, heart transplant patients, and severe or partially repaired cyanotic congenital heart defects.
A previously healthy term infant suddenly develops respiratory distress on the day 3 of life. An echocardiogram reveals coarctation of the aorta. Which of the following is the most appropriate treatment for immediate stabilization of this infant?

A. Digoxin
B. Furosemide
C. Albuterol
D. Racemic epiniphrine
E. Prostaglandin therapy
E. This infant’s symptoms began when his ductus arteriosus began to close. Prostaglandin therapy can reverse this process in the short-term. Surgery or catheterization techniques provide definitive repair.
A 12-year-old boy requires a sports physical examination. He denies chronic health problems, including adverse exertion symptoms. The clinician notes a I–II/VI left upper sternal border systolic murmur that does not radiate. The second heart sounds splits normally, and no audi- ble click is appreciated. Peripheral perfusion is normal, and the fingers are not clubbed. Which of the following is the best recommendation?

A. He should not play strenuous sports.
B. He can participate in sports without restrictions.
C. A chest radiograph and an ECG before further recommendation
can be made.
D. A cardiology evaluation.
E. He may participate in sports, but he should seek immediate medical attention for dyspnea or other adverse symptoms.
B. This child has a benign pulmonary flow murmur, differentiated from a pathologic pulmonary murmur in that it does not radiate, no click is heard, and no signs and symptoms of cardiac disease (digital clubbing, cyanosis, exercise intolerance) are found.
A term, 3700-g infant is born vaginally without complications. At 24 hours of age, a II/VI systolic murmur is noted in the mitral area that radiates to the back. A similar murmur is noted in the right axilla. The infant is pink and breathing easily, and his bedside chart shows that he has been taking 30 cc of formula approximately every 2 hours. Initial management should include which of the following?

A. Chest radiography, ECG, and four extremity blood pressures.
B. Immediate administration of prostaglandin E1.
C. Transfer to a neonatal intensive care unit.
D. Consultation by a pediatric cardiologist.
E. Discharge home with follow-up in the pediatric clinic at 3days of life.
E. This infant has peripheral pulmonic stenosis, a benign childhood murmur. Other frequently encountered benign childhood murmurs are the venous hum (a low-pitched murmur heard at the sternal notch only when the child is upright) and the Still vibratory murmur (a high-pitched “musical” systolic murmur heard best at the left ster- nal border in the supine position). Although it may be difficult to diagnose the multitude of pathologic heart sounds, clinicians cer- tainly should know the characteristics of the common benign childhood murmurs.
A 4-year-old boy presents for a well-child visit. His mother notes that he breathes fast and his lips turn “dusky” when he runs or plays hard. The symptoms resolve once he stops the activity. On examination, he has a II/VI left upper sternal border systolic murmur that radiates to the back; a faint click is heard. Which of the following is the most likely cause of this child’s exercise intolerance?

A. Asthma
B. Atrial septal defect
C. Pulmonary valve stenosis
D. Tricuspid atresia
E. Ventricular septal defect
C. Although pulmonary stenosis and tricuspid atresia are cyanotic heart lesions, exercise-induced cyanosis and systolic murmur are characteristic of pulmonary stenosis.
A 15-month-old girl is playing quietly in your waiting room. The skin around her mouth is faintly blue, but she appears comfortable. She arises from her squatting position to run after her brother, and she suddenly becomes dyspneic and cyanotic. She returns to a squatting position and soon is breathing comfortably with only slight perioral cyanosis. Which of the following would you expect to see on her chest radiograph?

A. A “boot-shaped” heart
B. An“eggonastring”
C. Lung hyperinflation
D. Pneumonia
E. Pulmonary congestion
A. This child has TOF; she experiences improvement when squatting and “tet” (hypercyanotic) spells when running. The “boot-shaped” heart is a characteristic chest radiographic finding.
You are called to the bedside of a mother who just delivered a healthy term infant and has a question regarding her infant’s nutrition. The mother was fed goat’s milk as a child and wants to do the same for her infant. Under which of the following conditions is goat’s milk accept- able as infant nutrition?

A. Goat’s milk proteins are hydrolyzed before feeds.
B. Infants are provided supplemental vitamins and minerals.
C. Goat’s milk is freshly obtained from goats.
D. Infants of mothers with milk intolerance should preferentially
receive goat’s milk.
E. Goat’s milk is diluted with water.
B. Infants drinking goat’s milk must have nutritional supplementa- tion with vitamin B12, folate, and iron. Several goat’s milk–based for- mulas including these nutrients are available. Fresh, unpasteurized goat’s milk can contain Brucella ovis and cause brucellosis. Diluting milk will only serve to dilute the caloric content.
You receive the results of a CBC you performed in your clinic on a pal- lorous 9-month-old boy. Other than pallor, no historical or physical examination concerns were noted during the patient’s visit. The labora- tory technician reports a hemoglobin of 8.6 g/dL, an MCV of 105 fL, and platelet count of 98,000/mm3. You are also told that the white blood cell count is 8500/mm3 and the differential reveals 47% neutrophils and 42% lymphocytes, and that no atypical lymphocytes are seen. Which of the following is the most appropriate next step in this child’s care?

A. Measurement of serum iron and total iron binding capacity levels.
B. Initiate oral iron supplementation.
C. Measurement of vitamin B12 and folate levels.
D. Begin oral vitamin B12 and folate supplementation.
E. Obtaina STAT referral to pediatric hematologist.
C. This infant has hematologic parameters consistent with macrocytic anemia. The mild thrombocytopenia reported is periodically seen in patients with vitamin B12 deficiency, and is thought to be related to impaired DNA synthesis and ineffective thrombopoiesis. The results reported are not typical for iron deficiency and neither an iron panel nor iron supplementation is warranted. At this point, your workup should include checking folate and B12 levels; supplementation of these compounds is not yet justified. Myelodysplasia or leukemia is in the differential, but is probably less likely with a normal white blood cell count and differential (no atypical cells); referral to Pediatric Hematology may ultimately be required, but some prelimi- nary data can be gathered first.
The parents of a previously healthy 3-year-old girl bring the child to your office because she is complaining that her tongue hurts. The par- ents also report she has appeared weak and listless over the last several months, and has not been eating well. Recently she has exhibited trouble walking. The family usually eats a regular diet, including meats and vegetables. On physical examination, her tongue is smooth, red, and tender. She is pale and tachycardic. Her complete blood count reveals a macrocytic anemia. Which of the following is the most likely diagnosis in this child?

A. Folate deficiency
B. Irondeficiency
C. Vitamin D deficiency
D. Zinc deficiency
E. Vitamin B12 deficiency
E. This is the typical presentation for juvenile pernicious anemia, a rare autosomal recessive condition in which the child is not able to secrete intrinsic factor and cannot absorb vitamin B12. Supplies of vitamin B12 passed to the fetus from the mother typically are sufficient for at least the first 1 to 2 years of life. A deficiency in transcobalamin results in megaloblastic anemia in infancy because transcobalamin is required for B12 transport and utilization; therefore, vitamin B12 pro- vided by the mother cannot be used effectively.
You are working at a Native American clinic in Alaska. A 16-year-old adolescent female comes to your office for an evaluation of lethargy. Her father notes that recently she has looked pale. She eats a regular diet and has no significant past medical history. Her menses are regu- lar and have not been excessive. During the last few years, she has helped her mother in the family seafood restaurant after school, but is increasingly tired and unable to complete all of her work. Her com- plete blood count reveals a megaloblastic anemia. Which of the fol- lowing is the next appropriate study?

A. Folate level
B. Stool for rotavirus
C. Iron level
D. Stool for ova and parasites
E. Transcobalamin level
D. The fish tapeworm Diphyllobothrium latum uses vitamin B12 for growth and egg production; as many as one million eggs per day may be produced. The parasite also inactivates the vitamin B12–intrinsic factor complex, inhibiting absorption in the terminal ileum. The fish tapeworm is the longest tapeworm to infect humans, sometimes growing to more than 10 m in length. Most infestations are asymp- tomatic, with megaloblastic anemia occurring in 2% to 9% of tape- worm infections. Risk factors include eating raw or undercooked fish. In North America, it is most commonly seen in the northern United States, Alaska, and Canada. Eggs have a unique morphology and are easily found in stool samples.
A 3650-g term infant has ambiguous genitalia, including an enlarged clitoris/microphallus and one palpable testis in the labioscrotal folds. Sonogram reveals a uterus and ovaries. Which of the following is the most likely explanation for the child’s ambiguous genitalia?

A. Aromatase deficiency
B. Congenital adrenal hyperplasia
C. Female pseudohermaphroditism
D. Male pseudohermaphroditism
E. True hermaphroditism
E. The gonad in the labioscrotal fold suggests a testis, but a uterus and an ovary on sonography are highly suggestive of a true hermaphrodite. Gender assignment in this case should be based on the possibility of surgical correction of the external genitalia. Assignment of female sex and an attempt to preserve ovarian tissue is appropriate.
A mother brings in her 1-week-old son who has vomited four times over the last 24 hours. He has no fever or diarrhea. The infant is breast-feeding poorly and is “floppy” per the mother. He has had only one wet diaper in the last 12 hours. Physical examination reveals a lethargic infant who has lost 250 g since birth, with pulse of 110 bpm, dry oral mucosa, and no skin turgor. Which of the following tests would be reasonable to consider after stabilization and electrolyte measurement?

A. Serum cortisol level
B. Urinecortisollevel
C. Serum 21-hydroxylase level
D. Serum 17α-hydroxyprogesterone level
E. Serum testosterone level
D. Male infants with salt-losing CAH develop clinical symptoms similar to pyloric stenosis, intestinal obstruction, heart disease, cow’s milk intolerance, and other causes of failure to thrive. Their genitalia appear normal. A serum 17α-hydroxyprogesterone level typically is elevated. Without appropriate treatment (hydrocortisone, mineralo- corticoid, and sodium supplementation), cardiovascular collapse and death may occur within a few weeks. Many states have neonatal screening programs for CAH, yet infants with salt-losing CAH (21- hydroxylase deficiency) can become very ill and die before the screening results are known.
A mother brings in her 15-year-old daughter because she has never started her periods. She otherwise is healthy and takes no medications. Her past medical history is unremarkable except for inguinal hernia repair as an infant. Family history is unremarkable. She is at the 75th percentile for height and weight, has Tanner stage IV breast develop- ment, and no pubic or axillary hair development. Her anogenital examination reveals a short, pocketlike vaginal opening. Which of the following is the most likely explanation for her amenorrhea?

A. Adrenal tumor
B. Congenital adrenal hyperplasia
C. Pituitary tumor
D. Testicular feminization
E. Turner syndrome
D. Testicular feminization results from decreased androgen binding to target tissues or androgen insensitivity. Patients have 46,XY karyotypes yet appear as phenotypically normal females with a short or atretic vagina. Androgen insensitivity is the most common form of male pseudohermaphroditism. Maintaining female gender assignment is appropriate, and vaginoplasty is frequently needed after puberty.
You examine a full-term 3780-g newborn in the nursery and notice that he has marked hypotonia, a very small penis, and unilateral cryp- torchidism. Which of the following is the most likely explanation for these findings?

A. Congenital adrenal hyperplasia
B. Male pseudohermaphroditism
C. Maternal treatment with steroids
D. Mixed gonadal dysgenesis
E. Prader-Willi syndrome
E. Although severe hypotonia, failure to thrive, and hypogonadism characterize Prader-Willi syndrome in early life, hyperphagia, obesity, mental retardation, and the appearance of bizarre behavior manifest by age 6 years. Morbid obesity, limited sexual function, and severe behavioral abnormalities may occur. Mixed gonadal dysgenesis is a reasonable choice given the unilateral cryptorchidism and hypogo- nadism, but severe hypotonia usually is not a finding in that disorder.
A 13-year-old boy has a 1-day history of fever and lethargy, and was unable to be awoken this morning. In the emergency department his respiratory rate is 7 breaths/min, heart rate 55 bpm, temperature 105.8°F (41°C), and blood pressure (BP) 60/40 mm Hg. He has altered mental status, a stiff neck, and a purpuric rash over his trunk. Which of the following is the most appropriate next step in the management of this patient?

A. Computerized tomography of the head
B. Intravenous antibiotics
C. Intubation
D. Lumbar puncture
E. Serum chemistries
C. This patient in the question has meningococcemia; he is in shock, and he is about to die. The ABCs of airway, breathing, and circulation should always take precedence over diagnostic studies. N meningitidis can present as meningococcemia with purpura and shock; in some cases patients will also have meningitis. The LP should be deferred, however, until he is clinically stable. Intravenous fluids to support his cardiovascular status and antibiotics should be administered immediately.
An 8-year-old girl has persistent fever and headaches. Her parents report that for the 2 weeks prior she has complained of frontal headache that was significant enough to keep her from school. She has had intermittent temperature elevations to 101°F (38.3°C), and she started vomiting a nonbloody, nonbilious fluid a few days ago. She has had frequent otitis media and sinusitis episodes, but her last episode of otitis media occurred approximately 5 weeks ago. On examination, you find a lethargic girl in no respiratory distress. She has a tempera- ture of 100°F (37.7°C), heart rate 109 bpm, and blood pressure (BP) 100/60 mm Hg. She has nuchal rigidity and frontal sinus tenderness. Which of the following is the most appropriate next step in the man- agement of this patient?

A. Computerized tomography of the head
B. Intravenous promethazine for emesis
C. Lumbar puncture
D. Sinus radiographs
E. Trial of subcutaneous sumatriptan for migraine
A. This girl’s history of sinusitis and a prolonged headache with worsening emesis and nuchal rigidity suggest an intracranial abscess due to her sinusitis. In her case, CNS imaging (with contrast) is per- formed prior to an LP. Performing an LP when a mass lesion might be causing increased intracranial pressure can result in herniation of the brain and patient death. Sinus films would show sinusitis but would not reveal an intracranial abscess. Merely treating her symptoms with promethazine or sumatriptan would delay the diagnosis of her underlying problem.
A 2-week-old infant develops a temperature to 102°F (38.9°C). Pregnancy and delivery were uncomplicated. The irritable, fussy infant has a heart rate of 170 bpm and respiratory rate 40 breaths/min. The anterior fontanelle is full, but he has no nuchal rigidity; the rest of the examination is unre- markable. Which of the following is the most appropriate management of this infant?

A. Encourage oral fluids and office follow-up in 24 hours.
B. Order computerized tomography of thehead followed by an LP.
C. Perform an LP, blood culture, urine culture, and admit to the hospital.
D. Prescribe intramuscular ceftriaxone and office follow-up in 1week.
E. Prescribe oral amoxicillin and office follow-up in 1 week.
C. This infant potentially has a serious bacterial infection, and an evaluation including a LP is performed. Infants do not reliably demon- strate a Kernig or Brudzinski sign; a lack of nuchal rigidity should not preclude an LP. Computerized tomography scan before an LP in an infant with an open anterior fontanelle is rarely necessary; brain her- niation is exceedingly rare. A course of oral antibiotics, or a single dose of ceftriaxone, is not sufficient to treat meningitis or septicemia.
A 14-year-old boy complains of fever and stiff neck for 2 days. He has a sore throat and has been unable to eat for 1 day because of the pain. On examination, he is alert and oriented, but he has nuchal rigidity and posterior pharyngeal midline fullness. He drools to avoid the pain of swallowing. Which of the following is the best next step in the management of this patient?

A. Order computerized tomography of the head.
B. Order lateral neck radiographs.
C. Perform a lumbar puncture.
D. Prescribe intramuscular antibiotics.
E. Prescribe oral antibiotics.
B. A retropharyngeal abscess is causing this boy’s neck stiffness; he does not have meningitis. He has a normal mental status, dysphagia, and fullness in his oropharynx. Lateral neck films are a simple way to confirm this diagnosis. Prescribing antibiotics without identifying the diagnosis would not be appropriate in this case.
You are the team physician for a high-school football team. During the first quarter of the latest game, you watch as your star quarterback is sacked with a helmet-to-helmet tackle. He does not get up from the initial impact. You sprint onto the field and assess the injured player. He is breathing and has a steady pulse, but he is unconscious. As you begin your evaluation, he wakes up. He remembers his name but can- not remember the day, his position on the team, or how he got to the game. He has no sensory or motor deficit suggestive of a cervical spine injury, and you assist him off the field. After 10 minutes he is fully ori- ented (although he still cannot remember what he had for breakfast) and wants to go back in. The coach tells him he is sitting out for the rest of the game. The player appeals to you. Which of the following is the most appropriate management?

A. Affirm the coach’s decision. Tell the player that he will need sequential evaluations before he can come back to practice.
B. Affirm the coach’s decision.Tell the player he can come back and practice tomorrow.
C. Refute the coach’s decision. Tell the player he can resume playing now.
D. Refute the coach’s decision. Tell the player he can resume playing after half-time.
E. Strap the player to a backboard and take him to the hospital.
A. Although controversial, the correct answer is for a player who sustains a concussion resulting in loss of consciousness to refrain from play for the remainder of the day. The most recent clinical report from the American Academy of Pediatrics concerning condi- tions affecting sports participation references the 2nd International Conference on Concussion in Sports from 2004. This report suggests that individualized and frequent reassessment over time, and a step- wise return to play, is more useful than a predetermined length of time to refrain from additional sports.
A 17-year-old adolescent female is brought to the hospital after a motor vehicle crash. She and her boyfriend had been drinking beer and were on their way home when she lost control of the car and hit the side wall of the local police station. She reportedly had a brief loss of conscious- ness but currently is oriented to name, place, and time. She responds appropriately to your questions. While waiting for her cervical spine series, she vomits and lapses into unconsciousness. She becomes brady- cardic and develops irregular respirations. Which of the following brain injuries is most likely in this case?

A. Subdural hemorrhage
B. Epidural hemorrhage
C. Intraventricular hemorrhage
D. Posttraumatic epilepsy
E. Concussion
B. This teen displays the typical adult course of epidural hemorrhage (an initial period of altered mental status [initial concussion], a period of lucidity, and then redevelopment of altered mental status and symptoms of increased ICP [hematoma effect]). Younger children typically do not display this pattern. Immediate neurosurgical evaluation is required.
Several days after emergent management, the adolescent in question 39.2 is transferred to your general inpatient ward service from the inten- sive care unit. She is concerned about her prognosis. Which of the follow- ing statements is correct?

A. She will need extensive neuropsychiatric evaluation before she can return to school.
B. She will likely have headaches, fatigue, nausea, and sleep distur- bances.
C. She will likely develop seizures and needs 2 years of prophylactic seizure medicine.
D. She can no longer be legally permitted to drive because she has had brain surgery.
E. She should have few long-term problems.
E. The acute epidural hemorrhage mortality rate is higher than that of acute subdural hemorrhage, but long-term morbidity is less. The com- plaints in answer B are common after a subdural hemorrhage. A seizure disorder may preclude driving; a cranial surgery history does not.
A 7-month-old child presents to the emergency room after reportedly falling from his high chair. The parents report no loss of consciousness, other trauma, or medical problems. Your examination reveals a few old bruises but no evidence of acute trauma or fracture. He is irritable, so you request a CT scan of the brain without contrast. The pediatric radiologist reports bilateral frontal subdural hematomas and notes two healing skull fractures that she estimates to be approximately 2 weeks old. Which of the following is the best next step in this child’s management?

A. Observe him for 6 hours in the emergency center.
B. Assess bleeding time and prothrombin time.
C. Order magnetic resonance imaging of the head.
D. Discharge him from the emergency center with head injury
precautions.
E. Order an electroencephalography and a neurology consultation.
C. This child has evidence of old skull fractures with subdural hematomas. Head magnetic resonance imaging would help to deter- mine the hematoma age. If the hematoma blood age correlates with the estimated skull fracture age, child abuse is considered. Neurology may be helpful later, but an immediate consultation would be of lim- ited benefit before additional data were gathered. Discharge with the information presented in the case is dangerous; the child likely requires hospital admission and the involvement of social services. Bleeding studies are unlikely to be helpful; the child has no history consistent with a bleeding disorder, nor would a bleeding disorder explain the old fractures.
A 10-day-old infant has a painful, red vesicular rash in the diaper area. He is mildly fussy but afebrile, and he has good oral intake. Which of the following is the most appropriate management of this infant?

A. Hospitalize the patient, obtain HSV cultures, and initiate par-
enteral acyclovir.
B. Order an EEG and brain MRI stat.
C. Perform a Tzanck smear and send the patienthome if it is negative.
D. Prescribe an antifungal cream and follow up by telephone in 24 hours.
E. Schedule an appointment witha pediatric dermatologist.
A. In contrast to older children and adults, infants with suspected herpes skin lesions require parenteral antiviral therapy to prevent more serious sequelae.
A woman presents for her first prenatal visit at 9-week gestation. She reports that she is generally healthy, except that she has an outbreak of genital herpes approximately once per year. To prevent transmission of the virus to her infant, her physician should do which of following?

A. Anticipate a cesarean section delivery.
B. Order titers to determine if the infection is HSV-1 or HSV-2.
C. Perform weekly genital viral cultures starting at 36-week gestation.
D. Perform a cesarean delivery if herpetic lesions or prodromal symp-
toms are present when labor has begun.
E. No change in management is indicated; the risk of infant trans-
mission is low even if she has an outbreak at delivery.
D. Even though the viral transmission risk in the setting of a recur- rent HSV outbreak is low, cesarean section is indicated if lesions are present at the time of delivery. Surveillance cultures are not recom- mended; negative results a few days prior to delivery do not preclude a later outbreak, and results of analysis of a more recently obtained specimen may not be available.
A 5-year-old with dysuria is found on examination to have herpetic gen- ital lesions. Which of the following is the best next step in management?

A. Ask the parent to leave the room and then ask the girl in an open-
ended fashion whether she has ever been inappropriately touched
in her private area.
B. Prescribe oral acyclovir and ask her to follow up in 2 days. C. Admit her to the hospital for parenteral antiviral therapy.
D. Ask how often the mother has outbreaks of genital herpes.
E. Send a urine culture and have the mother apply petroleum jelly until the lesions heal.
A. The possibility of sexual abuse is considered in a child who pres- ents with genital herpes beyond the neonatal period. It is important to know who helps to bathe the child, and whether these persons have ever had herpes, as nonsexual transmission is also possible.
The results of PCR of CSF from a 15-year-old adolescent male with encephalitis demonstrate an HSV infection. His parents ask about his prognosis. Which of the following is likely to be true?

A. He will most likely die.
B. He will likely survive, but will certainly have serious neurologic impairment.
C. Most children with HSV encephalitis survive; many (but not all) are left with some permanent neurologic deficits.
D. They should consider placing him in a long-term care facility upon discharge.
C. Although the majority of children with HSV encephalitis suffer permanent neurologic impairment, good outcomes are possible with appropriate medical and rehabilitative therapy.
A 2-hour-old term newborn male has coughing, choking, and cyanosis prior to feeding. A nasogastric tube is placed and meets resistance at 10 cm. Prenatal history is significant for polyhydramnios. Which of the following is most likely to be found in this infant?

A. Congenital cataracts
B. Gingivalhyperplasia
C. Hepatosplenomegaly
D. Microcephaly
E. Fusion of two lower thoracic vertebral bodies
E. The infant probably has esophageal atresia. VATER association, as described in the case, can have vertebral anomalies such as fused or bifid vertebral bodies. None of the other findings listed is com- monly associated with VATER.
An infant with a history of recurrent pneumonia is diagnosed with TEF at 8 months of age. Which of the following statements is correct?

A. The infant most likely has a “H-type” TEF.
B. The infant most likely has proximal esophageal atresia with distal fistula.
C. The infant likely has a previously undetected, associated finding of imperforate anus.
D. The infant is unlikely to have gastroesophageal reflux.
E. The infant is likely to have cystic fibrosis.
A. This infant likely has an H-type TEF, found later in infancy with recurrent pneumonias and/or feeding difficulty. Patients with esophageal atresia and distal fistula present in the first hours of life because of their inability to swallow oropharyngeal secretions. Infants with imperforate anus also present as neonates. All patients with TEF are at high risk for gastroesophageal reflux.
A 2-year-old girl with a history of esophageal atresia and a ventricular septal defect is hospitalized with Pneumocystis carinii pneumonia. Her immunodeficiency is likely a result of which of the following?

A. Bruton agammaglobulinemia
B. Chronic granulomatous disease
C. DiGeorge syndrome
D. Hyperimmunoglobulin E syndrome E. Severe combined immunodeficiency syndrome
C. DiGeorge syndrome (thymic hypoplasia) results from abnormal third and fourth pharyngeal pouch formation during fetal develop- ment. Neighboring structures formed during the same fetal growth period are often affected. Associated conditions include anomalies of the great vessels, esophageal atresia, bifid uvula, congenital heart dis- ease, short philtrum, hypertelorism, antimongoloid slant palpebrae, mandibular hypoplasia, and low-set, notched ears. DiGeorge syndrome may present in neonates as hypocalcemic seizures because of parathyroid hypoplasia.
A 2-year-old boy, living with new foster parents for 3 weeks, has become progressively short of breath. When he first arrived at their home, he was active and playful, but now he is too tired to play. They have few details, but they know that he had neonatal surgery for a problem with his “esophagus being connected to his lungs” and that he takes no medications. On examination, he is afebrile, diaphoretic, tachycardic, and tachypneic. His symptoms can most likely be attrib- uted to which of the following?

A. Adjustment disorder
B. Heart failure secondary to ventricular septal defect
C. Kawasaki disease
D. Reactive airway disease
E. Rheumatic heart disease
B. This child likely had undergone TEF repair and has associated congenital heart disease with heart failure symptoms.
A term male is born to a 33-year-old woman who had little prenatal care. Immediately after birth he has cyanosis and respiratory distress. Chest auscultation in the delivery room reveals right-sided heart sounds and absent left-sided breath sounds. Which of the following is the most appropriate next step?

A. Assess the abdomen to evaluate for possible congenital diaphragmatic hernia.
B. Order a computerized tomography of the chest.
C. Order ultrasonography of the chest.
D. Perform a needle thoracostomy for possible pneumothorax.
E. Prepare the infant for ECMO.
A. Evaluation of neonates born with respiratory distress and unilat- eral breath sounds includes an abdominal examination. With asym- metrical breath sounds, pneumothorax and CDH are considered. This infant’s scaphoid abdomen suggests CDH; needle thoracostomy is avoided because intestinal perforation may occur. The patient is stabilized and the need for ECMO is ascertained after the infant’s initial therapy response is evaluated.
A term female is born via repeat cesarean section to a 30-year-old woman. Immediately after birth she has mild respiratory distress. Chest auscultation in the delivery room reveals clear breath sounds. Which of the following is the most appropriate next step?

A. Endotracheal intubation with direct suction.
B. Begin intravenous antibiotic therapy.
C. Deliver surfactant therapy.
D. Observe and administer supplemental oxygen as needed.
E. Bag-and-mask ventilation.
D. As this infant most likely has TTN the next step is to observe and administer supplemental oxygen as needed.
A term male is born vaginally to a 22-year-old primigravida woman; the pregnancy was uncomplicated. Just prior to delivery, fetal brady- cardia was noted, and at delivery thick meconium is found. The infant has hypotonia and bradycardia. Which of the following is the first step in resuscitation?

A. Administration of epinephrine via endotracheal tube
B. Bag-and-mask ventilation
C. Endotracheal intubation with direct suction
D. Oxygen delivered by cannula in close proximity to the nares
E. Tracheostomy
C. Endotracheal intubation with direct suction is performed in a depressed infant with thick meconium noted at delivery. Bag-and- mask ventilation or endotracheal intubation without suction may increase the volume of meconium aspirated.
After the infant discussed in Question 48.3 is stabilized and admitted to the neonatal intensive care unit, a chest radiograph reveals bilateral patchy infiltrates with coarse streaking and flattening of the diaphragm. He abruptly has an increased oxygen requirement. Physical examination reveals decreased right-sided breath sounds. Which of the following is an accurate statement?

A. High positive end-expiratory pressure (PEEP) is useful in this condition.
B. Needle thoracostomy is contraindicated.
C. Chest radiography is likely to reveal CDH.
D. The infant is likely to have hyperresonance of the chest on the left
side.
E. Transillumination of the chest is likely to transmit excessive light
on the right side.
E. This infant likely has a right-sided pneumothorax; excessive light transmission by transillumination and right-sided hyperresonance with auscultation are expected. Infants with meconium aspiration and respiratory distress are at higher risk for pneumothorax, especially if high PEEP is used for oxygenation. A chest tube for the pneu- mothorax may be needed. Infants with severe respiratory distress or circulatory involvement may require emergent needle aspiration.
A 3-month-old boy presents with fever without a source. As part of his evaluation a urinalysis is performed; a UTI is suspected. Which of the following is the best next step?

A. If the urine culture reveals UTI, renal USG and VCUG should be performed.
B. VCUG should be performed only after a second UTI is diagnosed
C. Antibiotics should be initiated after urine culture and sensitivities are obtained.
D. Renal biopsy should be performed.
E. Preferred methods of collection for urine culture for this infant
include midstream clean-catch and bag urine.
A. For any infant male with a UTI, evaluation of anatomy and function is necessary. The preferred methods of urine collection include bladder catheterization and suprapubic bladder aspiration. Antimicrobial ther- apy is started empirically while awaiting urine culture and sensitivity results.
A 2-month-old girl presents with fever and vomiting. Her serum white blood cell (WBC) count is elevated. Urinalysis reveals 100 WBC per high-power field (unspun); it is positive for nitrates and leukocyte esterase. Urine culture results confirm a UTI; renal USG and VCUG show mild hydronephrosis and grade II VUR on the right. Which of the following is the best next step?

A. She will require surgical reimplantation of her right ureter.
B. Antimicrobial prophylactic therapy should be started when her
current course of antibiotics is completed.
C. VCUG should be performed on a monthly basis.
D. Subsequent urine specimen must be obtained only by suprapubic
aspiration.
E. Renal arteriography is indicated.
B. Infants and children with VUR receive prophylactic antimicrobial therapy and close monitoring for infection with urinalysis and urine culture at 3- to 4-month intervals. Sulfamethoxazole-trimethoprim, trimethoprim alone, and nitrofurantoin are commonly used for antimicrobial prophylaxis, VUR is graded from I to V based on the degree of reflux. Higher-grade reflux is less likely to resolve sponta- neously and is more likely to result in renal damage.
A 6-month-old infant male presents to your clinic with an abdominal mass, which was discovered by his new foster mother during the child’s bath. On physical examination, you also find macroglossia and right-sided hemihypertrophy. This infant is likely to have which of the following?

A. Down syndrome with duodenal atresia
B. Alagille syndrome and biliary atresia
C. Beckwith-Wiedemann syndrome with Wilms tumor
D. Neurofibromatosis and abdominal neurofibromas
E. Zellweger syndrome and hepatomegaly
C. This infant with features of Beckwith-Wiedemann syndrome is at high risk for developing Wilms tumor, hepatoblastoma, and gonadoblastoma.
An 8-year-old boy presents with bedwetting 3 to 4 times per week for “as long as he can remember.” He has a strong urine stream, daytime urine continence, and no UTIs. His physical examination is normal. Which of the following is the most appropriate next course of action?

A. Urodynamic studies.
B. Reassurance; he has secondary nocturnal enuresis.
C. Use of enuresis alarm.
D. Desmopress in acetate can be administered every 6 hours to control
enuresis.
E. Behavior modification that includes punishment for wet nights and rewards for dry nights.
C. Nocturnal enuresis occurs in 15% of 5-year-olds with a resolution rate of 15% per year. Males are more frequently affected, and family history is common. Initial evaluation includes a history of wetting pat- tern, prior UTI, and developmental, social, and emotional history. Physical examination includes kidney palpation, neurologic examina- tion, and examination of the back looking for sacral dimple or hairy nevus. Some recommend urinalysis and culture to rule out occult infection. The enuresis alarm has a success rate of 70% to 90% and requires parental support. Pharmacologic interventions include night- time doses of imipramine or oral desmopressin acetate. Intransal for- mulations of desmopressin are no longer approved for the treatment of primary nocturnal emuresis. Following use of desmopressin acetate, fluid intake is restricted to avoid hyponatremia. Pharmacologic treat- ment usually is reserved for special occasions, such as when the child is sleeping over at a friend’s house, summer camp, and so on. Behavior modification does not include punishment.
A 3-month-old male infant has intermittent neck contortions and arch- ing. He was term at birth, with an uneventful prenatal course and delivery. He frequently spits up after feeding, and has had one episode of pneumo- nia. Which of the following is the best next step in management?

A. Begin gentle stretching of the sternocleidomastoid muscle.
B. Evaluate him for gastroesophageal reflux disease (GERD).
C. Refer him for orthopedic evaluation.
D. Obtain cervical spine radiographs.
E. Observe and, if the condition persists, refer him for orthopedic evaluation.
B. This infant most likely has GER with intermittent torticollis (Sandifer syndrome). He has a history of frequently spitting up and has had pneumonia (possibly aspiration), indicating he has GER. Sandifer syndrome infants have abnormal head posturing associated with reflux. The head movements are thought to occur in response to pain or to protect the airway.
A 5-month-old female infant presents with sudden onset of torticollis and facial grimacing, but otherwise she appears alert and interactive. She has been doing well and has gained weight for the last month after having been prescribed ranitidine and metoclopramide for GER disease. Which of the following statements is accurate?

A. She is likely having a partial-complex seizure and needs an electroencephalograph.
B. A lumbar puncture for cell count, glucose, and protein is warranted.
C. Measurement of serum electrolyte and glucose levels is unnecessary.
D. She is likely having a dystonic reaction to one of her medications.
E. A cervical spine magnetic resonance image is likely to show a congenital abnormality.
D. This infant has sudden onset of the dystonic features of torticollis and facial grimacing, most likely as a result of the metoclopramide. However, initial evaluation for seizures, including measurement of serum electrolyte, glucose, and calcium levels, is indicated. Diphenhydramine administration may rapidly reverse this drug-induced dystonia. An MRI is unlikely to demonstrate a cervical abnormality because the symptom onset was abrupt. Cerebrospinal fluid analysis as a first step likely will not result in determination of the cause of this type of torticollis.
A 4-year-old boy presents with torticollis, fever, sore throat, and diffi- culty swallowing but no drooling. He denies headache and dyspnea, and he remains only somewhat playful. Examination reveals posterior pharyngeal edema. Which of the following is the best next step in management?

A. Examine his cerebrospinal fluid. B. Obtain imaging studies of the airway and soft tissues of the neck.
C. Send a throat culture and begin antibiotic therapy based on the
results.
D. Begin oral penicillin.
E. Prescribe ibuprofen and neck stretching exercises.
B. This child has signs and symptoms of retropharyngeal cellulitis or abscess. Such patients may have fever, dysphagia, drooling, stiff neck, dyspnea, or airway stridor. Physical findings include midline or unilateral swelling that may become a fluctuant mass. Management includes antibiotic therapy with possible incision and drainage of the abscess. Computerized tomography may be helpful in early identification of abscess formation.
A 1-week-old female newborn presents with her new adoptive parents. The family complains that she seems to have a twisted neck. They know only that “delivery was almost a C-section because the baby was lying sideways.” She has been feeding well and has had appropriate urine and stool output for the last 24 hours. Physical examination is signifi- cant for torticollis. Which of the following statements is most accurate?

A. She is at significant risk for aspiration pneumonia.
B. The parents should immediately begin a regimen of gentle stretching of the neck.
C. Radiographs of the cervical spine should be obtained.
D. Immediate orthopedic consultation should be arranged.
E. Immediate neurologic consultation should be arranged.
C. This child appears to have had a difficult delivery, making mus- cular torticollis likely. If cervical spine radiography is normal, the parents can begin gentle stretching to move the head in a neutral position. If the condition persists, orthopedic referral is necessary.
A 6-month-old infant with right-sided dacryostenosis presents with mucopurulent discharge and an indurated, erythematous, tender 1-cm mass on the right side just below his nasal bridge. He has a tempera- ture of 101°F (38.3°C). Which of the following is the best next step in management?

A. Administer intravenous antibiotic therapy.
B. Begin a course of topical antimicrobial treatment.
C. Recommend massage and warm compresses to the affected area.
D. Incise and drain the area.
E. Refer the child for an outpatient ophthalmologic evaluation.
A. This infant has dacryocystitis and needs immediate systemic (not topical) antibiotics. Surgical treatment may be necessary.
An 8-hour-old newborn presents with bilateral conjunctivitis following routine newborn care in the nursery. Which of the following is the best next step in management?

A. Administer prophylaxis with topical erythromycin.
B. Send the eye discharge for culture and start antibiotics based on culture results.
C. Start saline eye washes.
D. Begin systemic antibiotic therapy with erythromycin.
E. Begin systemic antibiotic therapy with ceftriaxone.
B. Conjunctivitis in the first few hours of life is most likely caused by chemical irritation. Laboratory testing of the discharge is performed; treatment usually can be based upon laboratory results.
A 2-week-old newborn presents with his foster mother because he has bilateral purulent eye discharge. His prenatal history is unknown. The examination demonstrates significant tarsal conjunctivitis and eye dis- charge. Which of the following statements is most accurate?

A. Initial therapy includes administration of intramuscular ceftriaxone.
B. The organism likely responsible also causes pneumonia in 1- to
3-month-old infants.
C. Immediate referral to a pediatric ophthalmologist is warranted.
D. Warm compresses and gentle massage are first-line therapies.
E. Topical antimicrobial therapy is preferred.
B. A chlamydial infection is the most likely cause of this patient’s con- junctivitis. Chlamydia trachomatis causes infantile pneumonia, generally between 1 and 3 months of age, presenting with cough, tachypnea, and rales but no fever. A complete blood count might show eosinophilia. A 14- to 21-day course of oral erythromycin is given.
A 4-month-old infant has excessive right-sided tearing. His mother states he becomes irritable in bright light and calms in a darkened room. On examination, he has eye asymmetry, with the right eye appearing to be larger than the left. Which of the following statements is accurate?
A. Warm compresses and gentle massage are first-line therapy.
B. In most cases, treatment is nonsurgical.
C. The infant has the classic features of Down syndrome.
D. Immediate systemic antibiotic therapy will reduce complications.
E. Immediate referral to a pediatric ophthalmologist is warranted.
E. A history of excessive tearing and photophobia and examination findings of corneal enlargement suggest an immediate evaluation for congenital glaucoma is indicated.
A 26-month-old boy presents because of maternal concern about his hearing. Over the past few weeks, his mother has had to speak more loudly in order for him to respond. He has a greater than 50-word vocabulary and can put together 2- to 3-word sentences. Three weeks prior he had an upper respiratory infection (URI). Which of the fol- lowing is the best next step in treatment?

A. Order ABR testing.
B. Perform otoscopy with insufflation.
C. Send him for a complete audiologic evaluation.
D. Perform hearing screening in the office.
E. Explain to the mother that 2-year-old children often do not respond to their parents.
B. This child has normal speech development and was recently noted to have a possible hearing deficit. With the recent URI, he is at risk for otitis media with effusion and conductive hearing loss. Otoscopy with insufflation (gently blowing air into the ear canal to determine tympanic membrane movement) is helpful for qualitative evaluation of middle ear effusion. Tympanometry is a reliable, quantitative tool for assessing middle ear effusion. If he has conductive hearing loss, further evaluation is indicated.
A 4-month-old boy has a white forelock, a broad mandible, and lat- eral displacement of his inner canthi. His mother also has a white fore- lock. Which of the following statements is true?

A. A urinalysis will demonstrate increased protein levels.
B. He is not at risk for hearing loss if his mother has normal hearing.
C. He is at risk for SNHL; order an audiologic evaluation.
D. The inheritance pattern of this disorder is X-linked recessive.
E. He should have on going office hearing screening with referral for
formal hearing if abnormalities are detected.
C. This child has features of Waardenburg syndrome (partial albinism, often a white forelock, SNHL, lateral inner canthi displacement, medial eyebrow flaring, and a broad nasal bridge and mandible); inheritance is autosomal dominant. Children with syndromic features strongly associ- ated with hearing loss require hearing evaluation.
Which of the following groups of children is at especially high risk for hearing loss?

A. A full-term, large-for-gestational-age male born to a mother with gestational diabetes
B. An appropriate-for-gestational age (AGA) infant, the product of a 34-week pregnancy, who had Apgar scores of 7 and 8 at 1 and 5 minutes, respectively
C. A term, 3300-g infant born by cesarean section who had a peak total bilirubin level of 18 mg/dL at 72 hours of life
D. A term AGA baby who received cefotaxime and ampicillin for 48 hours for suspected sepsis
E. A term AGA infant born by cesarean section for placental abruption with Apgar scores of 3 and 5 at 1 and 5 minutes, respectively
E. Infants born with Apgar scores of 4 or less at 1 minute and 6 or less at 5 minutes require audiologic evaluation. Other infants who should undergo testing include those with a family history of childhood SNHL; cytomegalovirus, rubella, syphilis, herpes, or toxoplasmosis infection; craniofacial anomalies; birth weight less than 1500 g; hyper- bilirubinemia at a level requiring exchange transfusion; bacterial meningitis; mechanical ventilation for more than 5 days; and stig- mata of syndromes associated with hearing loss, especially those with renal abnormalities.
Which of the following would be the expected language development of a normal 24-month-old child?

A. Speech that is 90% understandable B. A 10-word vocabulary but no combination of words
C. A 50-word vocabulary and 2-word combinations to make a sentence D. Appropriate use of pronouns
E. A 200-word vocabulary and 4 to 5-word combinations to make a sentence
C. At 24 months of age, the average child has a vocabulary of approximately 50 words and forms 2-word sentences. A 12-month- old child has a vocabulary of two to four words in addition to appro- priately saying “mama” and “dada.” By 36 months, a child should have a vocabulary of 250 words, produce at least 3-word sentences, and use pronouns.
A 2-year-old boy presents 4 hours after a left arm injury. He tried to run into the street, and his mother held his left hand tightly and he fell. Since then he has not moved his arm. Now he holds the arm close to his body with the elbow flexed and the forearm pronated. The elbow is not erythematous or edematous. He cries when the elbow is touched. Which of the following is the best next step in management?

A. Obtain a radiograph of the left elbow.
B. Order a skeletal survey.
C. Place the left arm in a sling.
D. Supinate the child’s forearm while applying pressure over the radial head.
E. Apply traction to the forearm while increasing the degree of
pronation.
D. This child’s history is consistent with a traction injury involving an outstretched arm. The elbow is not swollen and the arm is held in a flexed and pronated position. The child likely has “nursemaid’s elbow.” To reduce the subluxation, apply radial head pressure while supinating the arm. If treatment is not delayed, he will usually begin using the arm promptly.
A 15-year-old adolescent female has 2 days of nasal congestion and cough. Upon auscultation of her back, you find the lesions noted (Figure 58–1). Which of the following is the most likely etiology for her condition?
A. Cupping
B. Physical abuse
C. Disseminated intravascular coagulation
D. Henoch-Schönlein purpura
E. Coining
A. This adolescent has multiple perfectly circular lesions on her back consistent with cupping; when asked, she gives the history of cup- ping. Physical abuse injuries likely would not be identical in appear- ance. Patients with DIC will have significant systemic manifestation, and the pattern of ecchymoses would not be symmetrical. Coining causes ecchymosis in a linear pattern.
Which of the following describes the most common form of child maltreatment?

A. Sexual abuse
B. Physical abuse
C. Neglect
D. Emotional abuse
E. Munchausen syndrome by proxy
C. The most common form of child maltreatment is neglect (failure to provide adequate nutrition, shelter, supervision, or health care).
A 4-month-old girl is fussy, appears to have pain on palpation of the right leg, and has bluish sclerae. Radiographs reveal a right femur frac- ture. Her parents deny any severe trauma but report she had multiple fractures as a child. Family history is also likely to include which of the following?

A. Blindness
B. Hearing loss
C. Tall stature
D. Renal disease
E. Aortic aneurysm
B. This infant has features of osteogenesis imperfecta, an autosomal dominant genetic disorder. Features include long bone fractures and vertebral injury with minimal trauma, short stature, deafness, and blue sclerae. Four types exist: type I is mild; type II is lethal (in utero or shortly thereafter); type III is the most severe; and type IV is mod- erately severe.
Malrotation with volvulus is most likely to be present in which of the following patients?

A. A healthy 15-month-old with severe paroxysmal abdominal pain and vomiting
B. A 15-year-old sexually active girl with lower abdominal pain
C. A 3-day-old term infant with bilious emesis, lethargy, and abdominal distension
D. A 4-day-old premature baby (33-week gestation) who has recently started nasogastric feeds; he now has abdominal distention, bloody
stools, and thrombocytopenia
E. A 7-year-old girl with abdominal pain, vomiting, fever, and diarrhea
C. The 3-day-old term infant with bilious emesis and abdominal dis- tension has classic presenting features of malrotation with volvulus. The 15-month-old child with paroxysmal abdominal pain is most likely to have intussusception. The adolescent female is evaluated for ectopic pregnancy, pelvic inflammatory disease, appendicitis, ovarian torsion, and ruptured ovarian cyst. The premature infant might have necrotizing enterocolitis, whereas the 7-year-old girl more likely has gastroenteritis.
A 3-day-old boy presents with 12 hours of bilious vomiting, abdomi- nal pain, and abdominal distension. Which of the following is the most appropriate next step in management?

A. Order an abdominal ultrasonography.
B. Order a computerized tomography scan of the abdomen.
C. Order a upper GI contrast series.
D. Order a barium enema.
E. Order a chest radiograph.
C. Order a upper GI contrast series. Fluid and electrolyte status should also be evaluated.
A 9-year-old boy has 24 hours of persistent abdominal pain and vomit- ing. His physical examination reveals abdominal guarding and right lower quadrant rebound tenderness. Which of the following is the most likely diagnosis?

A. Appendicitis
B. Gastroenteritis
C. Gastroesophageal reflux
D. Intussusception
E. Pyloricstenosis
A. This child most likely has appendicitis.
A previously healthy 18-month-old child has vomiting and severe, parox- ysmal, writhing abdominal pain (he prefers to have his knees flexed to the chest) alternating with periods of relative comfort with a soft, only mildly tender abdomen. On abdominal examination you find a sausagelike mass. He has not stooled, but you find blood upon digital rectal examination. Which of the following is the best next step in management?

A. Administer morphine for pain control.
B. Order a computerized tomography of the abdomen.
C. Obtain an air contrast enema.
D. Obtain serum acetaminophen levels.
E. Begin antibiotics for Escherichia coli 0157:H7.
C. The case describes the typical presentation of intussusception. Although a clinical diagnosis can be made, the diagnostic “gold” stan- dard and often treatment, is contrast enema. Air contrast usually is preferred because the complication risk is lower than with other forms of contrast material. Prior to diagnostic intervention, patients should undergo measurement of serum electrolyte and hemoglobin levels and receive fluid resuscitation. When suspicion for intussusception is high, a pediatric surgeon should be consulted. Classically described “currant jelly stools” are a late finding. Recurrence of intussusception following successful reduction occurs in 5% to 10% of cases.
A 6-week-old male infant has projectile emesis after feeding. He has an olive-shaped abdominal mass on abdominal examination. Which of the following statements is accurate?

A. He likely has hypochloremic metabolic alkalosis.
B. He likely has metabolic acidosis.
C. This condition is more common in female infants.
D. He should be restarted on feeds when the vomiting resolves.
E. He likely will develop diarrhea.
A. This infant has the features of pyloric stenosis, a condition four times more common in males and more common in first-born chil- dren. Affected infants usually present between the third and eighth week of life with increasing projectile emesis. Abdominal examina- tion may reveal an olive-shaped mass and visible peristaltic waves. Serum electrolyte levels usually reveal hypochloremic metabolic alka- losis. Ultrasonography is useful in confirming the diagnosis.